N453 Final: Practice Questions

Réussis tes devoirs et examens dès maintenant avec Quizwiz!

During the admitting neurologic examination, the nurse determines the patient has speech difficulties as well as weakness of the right arm and lower face. The nurse would expect a CT scan to show pathology in the distribution of the a. basilar artery. b. left middle cerebral artery. c. right anterior cerebral artery. d. left posterior communicating artery.

. left middle cerebral artery.

A patient is admitted after an ischemic stroke. She articulates well but does not understand what is being said to her. She has damage to: The thalamus The basal ganglia Broca's Area Wernicke's Area

Wernicke's Area

When caring for a patient with thrombocytopenia, the nurse instructs the patient to a. dab his or her nose instead of blowing. b. be careful when shaving with a safety razor. c. continue with physical activities to stimulate thrombopoiesis. d. avoid aspirin because it may mask the fever that occurs with thrombocytopenia.

a. dab his or her nose instead of blowing.

A nurse plans care for the patient with increased intracranial pressure with the knowledge that the best way to position the patient is to a. keep the head of the bed flat. b. elevate the head of the bed to 30 degrees. c. maintain patient on the left side with the head supported on a pillow. d. use a continuous-rotation bed to continuously change patient position.

b. elevate the head of the bed to 30 degrees.

You are called to the patient's room by the patient's spouse when the patient experiences a seizure. Upon finding the patient in a clonic reaction, what do you think you should do first? A. Turn the patient to the side. B. Start oxygen by mask at 6 L/min. C. Restrain the patient's arms and legs to prevent injury. D. Record the time sequence of the patient's movements and responses as they occur.

A. Turn the patient to the side.

Calculate the cerebral perfusion pressure (CPP) of a patient whose blood pressure (BP) is 106/52 mm Hg and ICP is 14 mm Hg. ________________________________________________ mm Hg .

56

The nurse is caring for a patient with type 1 diabetes mellitus who is admitted for diabetic ketoacidosis. The nurse would expect which laboratory test result? A. Hypokalemia B. Fluid overload C. Hypoglycemia D. Hyperphosphatemia

A. Hypokalemia

A 45-year-old woman is admitted with deep venous thrombosis and pulmonary emboli and is scheduled for insertion of an inferior vena cava filter. Her preoperative laboratory results show a platelet count of 25,000/mm3. Which nursing action is indicated? No action is necessary Start an extra intravenous line with a large-gauge catheter Notify the surgeon Monitor neurologic status carefully

Notify the Surgeon

Which drug treatment helps to decrease ICP by expanding plasma and the osmotic effect to move fluid? a. Dexamethasone b. Mannitol (25%) c. Oxygen administration d. Pentobarbital

b. Mannitol (25%)

A patient has sustained a diffuse axonal injury. An intraventricular catheter has been inserted, and the patient's intracranial pressure is 40 mm Hg. To maintain adequate cerebral perfusion pressure in this patient, the mean arterial pressure must be at least: 60 mmHg 80 mmHg 100 mmHg 150 mmHg

100 mmHg

A 52-year-old has been vomiting bright red blood for the past 24 hours. His hemoglobin level is 6 g/dL and hematocrit is 20%. Vital signs are blood pressure 90/60 mm Hg; heart rate 120 beats/min and regular; and respiratory rate 22 breaths/min and shallow. This patient receives a total of 12 units of packed RBCs. He also will need to receive: Potassium Whole blood Platelets Heparin

Platelets

Which factors decrease cerebral blood flow (select all that apply)? a. Increased ICP b. PaO2 of 45 mm Hg c. Arterial blood pH of 7.3 d. Decreased mean arterial pressure (MAP) e. PaCO2 of 30 mm Hg

a. Increased ICP d. Decreased mean arterial pressure (MAP) e. PaCO2 of 30 mm Hg

Number in sequence the events that occur in disseminated intravascular coagulation (DIC). Number the events from 1 through 7 starting with 1. Production of intravascular thrombin Uncompensated hemorrhage Fibrinogen converted to fibrin Inhibition of normal blood clotting Widespread fibrin and platelet deposition in capillaries and arterioles Release of fibrin-split products Activation of fibrinolytic system

1. Production of intravascular thrombin 2. Fibrinogen converted to fibrin 3. Widespread fibrin and platelet deposition in capillaries and arterioles 4. Activation of fibrinolytic system 5. Release of fibrin-split products 6. Inhibition of normal blood clotting 7. Uncompensated hemorrhage

A 72-year-old woman is admitted to the critical care unit in acute respiratory distress. Ventilation-perfusion scan results indicate a high probability of pulmonary embolism. Which of the following is the most likely location for the formation of a blood clot causing a pulmonary embolism? a. In the veins of the leg b. In the arteries of the leg c. In the left ventricle d. In the right ventricle

a. In the veins of the leg

A 37-year-old woman is admitted with a severe headache, nuchal rigidity, and facial asymmetry. She is difficult to awaken. Which of the following tests would be performed initially? Lumbar puncture Computed tomography (CT) scan of the head Angiogram Evoked potentials

Computed tomography (CT) scan of the head

A 50-year-old patient is admitted to the critical care unit after a rattlesnake bite. He is bleeding from his IV insertion sites and continues to seep blood from the wound. Which of the following diagnostic panels is suggestive of disseminated intravascular coagulation (DIC)? Decreased platelets, decreased fibrinogen, prolonged prothrombin time (PT), activated partial thromboplastin time (aPTT), prolonged thrombin time, increased fibrin (split) degradation products (FDPs) Increased platelets, increased fibrinogen, normal PT, aPTT, normal thrombin time, increased fibrin degradation (split) products (FDPs) Increased platelets, decreased fibrinogen, prolonged PT, aPTT, prolonged thrombin time, decreased fibrin degradation (split) products (FDPs) Decreased platelets, increased fibrinogen, normal PT, aPTT, normal thrombin time, increased fibrin degradation (split) products (FDPs)

Decreased platelets, decreased fibrinogen, prolonged prothrombin time (PT), activated partial thromboplastin time (aPTT), prolonged thrombin time, increased fibrin degradation (split) products (FDPs)

A patient presents to hospital and is diagnosed with diabetic ketoacidosis. Which respiratory pattern would you expect the patient to initially exhibit: Deep and rapid inspiration Deep and slow inspiration Shallow inspiration Rapid inspiration with slow exhalation

Deep and rapid inspiration

A patient is admitted with "the worst headache I've ever had." Computed tomography reveals an aneurysm in the circle of Willis. Following a clipping surgery, vasospasm may be treated with which of the following? Fluid administration Loop diuretics (e.g. Furosemide) Antifibrinolytic (aminocaproic acid) Osmotic diuretics (e.g., mannitol)

Fluid administration

A patient was admitted to the emergency department with status epilepticus. Which of the following drugs would most likely be prescribed initially to stop the seizures? Phenytoin Fosphenytoin Lorazepam Phenobarbital

Lorazepam

Match the acid-base imbalances with their common causes (answers may be used more than once). Renal Failure Septic Shock Diabetic Ketosis Respiratory Failure Prolonged Vomiting Sedative or Opioid Overdose Response to Anxiety, Fear, and Pain All Answer Choices A. Metabolic acidosis B. Respiratory acidosis C. Metabolic alkalosis D. Respiratory alkalosis

Renal Failure: A. Metabolic Acidosis Septic Shock: Metabolic Acidosis Diabetic Ketosis: Metabolic Acidosis Respiratory Failure: Respiratory Acidosis Prolonged Vomiting: Metabolic Alkalosis Sedative or Opioid Overdose: Respiratory Acidosis Response to Fear, Anxiety, or Pain: Respiratory Alkalosis

The nurse is caring for a patient after a head injury. How should the nurse position the patient in bed? Prone with the head turned to the right side High-Fowler's position with the legs elevated Supine position with the head on two pillows Side-lying with the head elevated 30 degrees

Side-lying with the head elevated 30 degrees

A patient with a head injury has an arterial BP of 92/50 mm Hg and ICP of 18 mm Hg. The nurse uses the assessments to calculate the cerebral perfusion pressure (CPP). How should the nurse interpret the results? The CPP is so low that brain death is imminent. The CPP is low, and the BP should be increased. The CPP is high, and the ICP should be reduced. The CPP is adequate for normal cerebral blood flow.

The CPP is low, and the BP should be increased.

A patient's eyes jerk while the patient looks to the left. The nurse will record this finding as a. nystagmus. b. CN VI palsy. c. ophthalmic dyskinesia. d. oculocephalic response.

a. nystagmus.

Of the following patients, the nurse recognizes that the one with the highest risk for a stroke is a(n) a. obese 45-yr-old Native American. b. 35-yr-old Asian American woman who smokes. c. 32-yr-old white woman taking oral contraceptives. d. 65-yr-old African American man with hypertension.

d. 65-yr-old African American man with hypertension.

A patient has the following arterial blood gas results: pH 7.52, PaCO2 30 mm Hg, HCO3− 24 mEq/L. The nurse determines that these results indicate a. metabolic acidosis. b. metabolic alkalosis. c. respiratory acidosis. d. respiratory alkalosis.

d. respiratory alkalosis.

Tangentiality

Wandering off topic or going off on tangents and never reaching the point

The patient at highest risk for venous thromboembolism (VTE) is a. a 62-year-old man with spider veins who is having arthroscopic knee surgery. b. a 32-year-old woman who smokes, takes oral contraceptives, and is planning a trip to Europe. c. a 26-year-old woman who is 3 days postpartum and received maintenance IV fluids for 12 hours during her labor. d. an active 72-year-old man at home recovering from transurethral resection of the prostate for benign prostatic hyperplasia.

a 32-year-old woman who smokes, takes oral contraceptives, and is planning a trip to Europe.

The patient has a lack of comprehension of both verbal and written language. Which type of communication difficulty does this patient have? a. Expressive aphasia b. Receptive aphasia c. Dysarthria d. Fluent dysphasia

b. Receptive aphasia

Risk factors to predict violent behavior

- A history of violence - single best predictor - Patients who are delusional, hyperactive, impulsive, or predisposed to irritability - Aggression by patients occurs most often in the context of "limit setting" by the nurse. - History of limited coping skills, including lack of assertiveness or use of intimidation, are at higher risk of using violence. - Situations that trigger impatience, irritation, or defensiveness.

The nurse is assessing the adaptation of a client to changes in functional status after a stroke (brain attack). Which observation indicates to the nurse that the client is adapting most successfully? 1. Gets angry with family if they interrupt a task 2. Experiences bouts of depression and irritability 3. Has difficulty with using modified feeding utensils 4. Consistently uses adaptive equipment in dressing self

4. Consistently uses adaptive equipment in dressing self

A client with a spinal cord injury is prone to experiencing autonomic dysreflexia. The nurse should include which measures in the plan of care to minimize the risk of occurrence? Select all that apply. 1. Keeping the linens wrinkle-free under the client 2. Preventing unnecessary pressure on the lower limbs 3. Limiting bladder catheterization to once every 12 hours 4. Turning and repositioning the client at least every 2 hours 5. Ensuring that the client has a bowel movement at least once a week

1. Keeping the linens wrinkle-free under the client 2. Preventing unnecessary pressure on the lower limbs 4. Turning and repositioning the client at least every 2 hours

The nurse is caring for a client who begins to experience seizure activity while in bed. Which actions should the nurse take? Select all that apply. 1. Loosening restrictive clothing 2. Restraining the client's limbs 3. Removing the pillow and raising padded side rails 4. Positioning the client to the side, if possible, with the head flexed forward 5. Keeping the curtain around the client and the room door open so when help arrives they can quickly enter to assist

1. Loosening restrictive clothing 3. Removing the pillow and raising padded side rails 4. Positioning the client to the side, if possible, with the head flexed forward

Which interventions are most appropriate for caring for a client in alcohol withdrawal? Select all that apply. 1. Monitor vital signs. 2. Provide a safe environment. 3. Address hallucinations therapeutically. 4. Provide stimulation in the environment. 5. Provide reality orientation as appropriate. 6. Maintain NPO (nothing by mouth) status.

1. Monitor vital signs. 2. Provide a safe environment. 3. Address hallucinations therapeutically 5. Provide reality orientation as appropriate.

The nurse is instituting seizure precautions for a client who is being admitted from the emergency department. Which measures should the nurse include in planning for the client's safety? Select all that apply. 1. Padding the side rails of the bed 2. Placing an airway at the bedside 3. Placing the bed in the high position 4. Putting a padded tongue blade at the head of the bed 5. Placing oxygen and suction equipment at the bedside 6. Flushing the intravenous catheter to ensure that the site is patent

1. Padding the side rails of the bed 2. Placing an airway at the bedside 5. Placing oxygen and suction equipment at the bedside 6. Flushing the intravenous catheter to ensure that the site is patent

A patient becomes anxious and says, "There are worms under my skin eating the hair follicles." How would you classify this assessment finding? A. Positive symptom B. Negative symptom C. Cognitive symptom D. Depressive symptom

A. Positive symptoms

The nurse is assigned to care for a client with complete right-sided hemiparesis from a stroke (brain attack). Which characteristics are associated with this condition? Select all that apply. 1. The client is aphasic. 2. The client has weakness on the right side of the body. 3. The client has complete bilateral paralysis of the arms and legs. 4. The client has weakness on the right side of the face and tongue. 5. The client has lost the ability to move the right arm but is able to walk independently. 6. The client has lost the ability to ambulate independently but is able to feed and bathe himself or herself without assistance.

1. The client is aphasic. 2. The client has weakness on the right side of the body. 4. The client has weakness on the right side of the face and tongue.

The nurse has given medication instructions to a client receiving phenytoin. Which statement indicates that the client has an adequate understanding of the instructions? 1. "Alcohol is not contraindicated while taking this medication." 2. "Good oral hygiene is needed, including brushing and flossing." 3. "The medication dose may be self-adjusted, depending on side effects." 4. "The morning dose of the medication should be taken before a serum medication level is drawn."

2. "Good oral hygiene is needed, including brushing and flossing."

A client with atrial fibrillation is receiving a continuous heparin infusion at 1000 units/hour. The nurse determines that the client is receiving the therapeutic effect based on which results? 1. Prothrombin time of 12.5 seconds 2. Activated partial thromboplastin time of 60 seconds 3. Activated partial thromboplastin time of 28 seconds 4. Activated partial thromboplastin time longer than 120 seconds

2. Activated partial thromboplastin time of 60 seconds Common laboratory ranges for activated partial thromboplastin time (aPTT) are 30 to 40 seconds. Because the aPTT should be 1.5 to 2.5 times the normal value, the client's aPTT would be considered therapeutic if it was 60 seconds. Prothrombin time assesses response to warfarin therapy.

The home health nurse visits a client at home and determines that the client is dependent on drugs. During the assessment, which action should the nurse take to plan appropriate nursing care? 1. Ask the client why he started taking illegal drugs. 2. Ask the client about the amount of drug use and its effect. 3. Ask the client how long he thought that he could take drugs without someone finding out. 4. Not ask any questions for fear that the client is in denial and will throw the nurse out of the home.

2. Ask the client about the amount of drug use and its effect. Rationale: Whenever the nurse carries out an assessment for a client who is dependent on drugs, it is best for the nurse to attempt to elicit information by being nonjudgmental and direct. Option 1 is incorrect because it is judgmental and off-focus, and reflects the nurse's bias. Option 3 is incorrect because it is judgmental, insensitive, and aggressive, which is nontherapeutic. Option 4 is incorrect because it indicates passivity on the nurse's part and uses rationalization to avoid the therapeutic nursing intervention.

The nurse is caring for the client with increased intracranial pressure. The nurse would note which trend in vital signs if the intracranial pressure is rising? 1. Increasing temperature, increasing pulse, increasing respirations, decreasing blood pressure 2. Increasing temperature, decreasing pulse, decreasing respirations, increasing blood pressure 3. Decreasing temperature, decreasing pulse, increasing respirations, decreasing blood pressure 4. Decreasing temperature, increasing pulse, decreasing respirations, increasing blood pressure

2. Increasing temperature, decreasing pulse, decreasing respirations, increasing blood pressure

The nurse is evaluating the status of a client who had a craniotomy 3 days ago. Which assessment finding would indicate that the client is developing meningitis as a complication of surgery? 1. A negative Kernig's sign 2. Absence of nuchal rigidity 3. A positive Brudzinski's sign 4. A Glasgow Coma Scale score of 15

3. A positive Brudzinski's sign

A client is taking the prescribed dose of phenytoin to control seizures. Results of a phenytoin blood level study reveal a level of 35 mcg/mL (140 mcmol/L). Which finding would be expected as a result of this laboratory result? 1. Hypotension 2. Tachycardia 3. Slurred speech 4. No abnormal finding

3. Slurred speech

The home health nurse visits a client who is taking phenytoin for control of seizures. During the assessment, the nurse notes that the client is taking birth control pills. Which information should the nurse include in the teaching plan? 1. Pregnancy must be avoided while taking phenytoin. 2. The client may stop the medication if it is causing severe gastrointestinal effects. 3. There is the potential of decreased effectiveness of birth control pills while taking phenytoin. 4. There is the increased risk of thrombophlebitis while taking phenytoin and birth control pills together.

3. There is the potential of decreased effectiveness of birth control pills while taking phenytoin.

A client recovering from a head injury is participating in care. The nurse determines that the client understands measures to prevent elevations in intracranial pressure if the nurse observes the client doing which activity? 1. Blowing the nose 2. Isometric exercises 3. Coughing vigorously 4. Exhaling during repositioning

4. Exhaling during repositioning

The nurse has instructed the family of a client with stroke (brain attack) who has homonymous hemianopsia about measures to help the client overcome the deficit. Which statement suggests that the family understands the measures to use when caring for the client? 1. "We need to discourage him from wearing eyeglasses." 2. "We need to place objects in his impaired field of vision." 3. "We need to approach him from the impaired field of vision." 4. "We need to remind him to turn his head to scan the lost visual field."

4. "We need to remind him to turn his head to scan the lost visual field."

A client has clear fluid leaking from the nose following a basilar skull fracture. Which finding would alert the nurse that cerebrospinal fluid is present? 1. Fluid is clear and tests negative for glucose. 2. Fluid is grossly bloody in appearance and has a pH of 6. 3. Fluid clumps together on the dressing and has a pH of 7. 4. Fluid separates into concentric rings and tests positive for glucose.

4. Fluid separates into concentric rings and tests positive for glucose.

The nurse is monitoring a hospitalized client who abuses alcohol. Which findings should alert the nurse to the potential for alcohol withdrawal delirium? 1. Hypotension, ataxia, hunger 2. Stupor, lethargy, muscular rigidity 3. Hypotension, coarse hand tremors, lethargy 4. Hypertension, changes in level of consciousness, hallucinations

4. Hypertension, changes in level of consciousness, hallucinations Rationale: Symptoms associated with alcohol withdrawal delirium typically include anxiety, insomnia, anorexia, hypertension, disorientation, hallucinations, changes in level of consciousness, agitation, fever, and delusions.

Clanging

A loose association using words chosen based on sound

When a patient returns to the clinical unit after an abdominal-perineal resection (APR), what should the nurse expect the patient to have? An abdominal dressing A temporary colostomy and drains An abdominal wound and drains A perineal wound, drains, and a stoma

A perineal wound, drains, and a stoma With an abdominal perineal-resection (APR), an abdominal incision is made, and the proximal sigmoid colon is brought through the abdominal wall and formed into a permanent colostomy. The patient is repositioned, a perineal incision is made, and the distal sigmoid colon, rectum, and anus are removed through the perineal incision, which may be closed or open and packed, and have drains.

An elderly patient with left-sided hemiplegia and a dilated right pupil states he cannot remember falling and has no signs of physical trauma. His family tells you he has been "acting differently" for approximately 1 month. Based on this information, he probably has: An epidural hematoma A subdural hematoma A subarachnoid hemorrhage An ischemic stroke

A subdural hematoma This patient probably has a chronic subdural hematoma. This is a venous bleed. Signs and symptoms may not occur for weeks to months after trauma. Many patients cannot remember a precipitating trauma, and it may occur spontaneously, especially in elderly patients, alcoholics, and patients taking anticoagulants.

The nurse obtains a blood pressure of 176/82 mm Hg for a patient. What is the patient's mean arterial pressure (MAP)? A. 113 B. 172 C. 211 D. 217

A. 113 Mean arterial pressure (MAP) = [Systolic + 2 (Diastolic)] / 3

Which assessment finding represents a negative symptom of schizophrenia? A. Apathy B. Delusion C. Motor tic D. Hallucination

A. Apathy

You administer the Confusion Assessment Method (CAM) tool to differentiate among various cognitive disorders, primarily because A. Delirium can be reversed by treating the underlying causes. B. Depression is a common cause of dementia in older adults. C. Nursing care should be based on the cause of the cognitive impairment. D. Drug therapy with antipsychotic agents is indicated in the treatment of dementia.

A. Delirium can be reversed by treating the underlying causes Rationale: Delirium, a state of temporary but acute mental confusion, is a common, life-threatening, and possibly preventable syndrome in older adults. Clinically, delirium is rarely caused by a single factor. It is often the result of the interaction of the patient's underlying condition with a precipitating event.

Which signs and symptoms are associated with acute stress disorder and often observed in patients who have been sexually assaulted? Select All that Apply A. Flashbacks B. Depression C. Auditory hallucinations D. Outburst of anger E. Amnesia for the event

A. Flashbacks B. Depression D. Outburst of anger E. Amnesia for the event

A priority goal of treatment for the patient with AD is to A. Maintain patient safety. B. Maintain or increase body weight. C. Return to a higher level of self-care. D. Enhance functional ability over time.

A. Maintain patient safety.

According to the Surviving Sepsis Campaign which of the following tasks should be accomplished within one hour of arrival to a healthcare facility? Select all that apply A. Measure lactate B. Administer broad-spectrum antibiotics, even if blood cultures have not been drawn C. Administer 30 mL/kg colloid fluids for hypotension D. Begin vasopressor therapy during or after fluid resuscitation to maintain MAP > 65 mmHg E. Re-measure lactate level if initial lactate is > 2 mmol/L

A. Measure lactate D. Begin vasopressor therapy during or after fluid resuscitation to maintain MAP > 65 mmHg E. Re-measure lactate level if initial lactate is > 2 mmol/L

Symptoms of withdrawal from opioids for which the nurse should assess include: A. Nausea, vomiting, diaphoresis, anxiety, and hyperreflexia. B. Mood lability, incoordination, fever, and drowsiness. C. Dilated pupils, tachycardia, elevated blood pressure, and elation. D. Excessive eating, constipation, and headache.

A. Nausea, vomiting, diaphoresis, anxiety, and hyperreflexia.

Which of the following treatments for an acute myocardial infarction is associated with the greatest patency of the infarct-related artery and less recurrent ischemia? A. Percutaneous coronary intervention (PCI) B. Aspirin C. Fibrinolytic drugs D. Nitrates

A. Percutaneous coronary intervention (PCI) PCI is the preferred treatment to reestablish patency of the infarct-related artery because it is more likely to attain greater patency with fewer complications and shorter length of stay than fibrinolytic drugs. Fibrinolytic drugs are used today if an interventional cardiac catheterization laboratory and/or interventional cardiologist are not available. Remember that if fibrinolytic drugs are used, they target only the clot and not the irregular surface on the intima (i.e., plaque) that triggered the clot.

Which of the following best describes multiple organ dysfunction syndrome (MODS)? A. Progressive insufficiency of two or more organ systems B. Cessation of function of two or more organ systems C. Sepsis involving two or more organ systems D. Loss of function of two or more components of the same organ system

A. Progressive insufficiency of two or more organ systems Patients may have sepsis with MODS, but MODS also may occur without sepsis. The definition of MODS does not require complete cessation of function, but two or more organs must be involved. Consider these words in the term: multiple so two or more organs, dysfunction rather than failure.

A patient undergoing alcohol rehabilitation decides to begin disulfiram therapy. Patient teaching should include the need to (Select all that apply) A. Read labels of all liquid medications B. Avoid aged cheeses C. Maintain an adequate dietary intake of sodium D. void alcohol-based products E. Wear sunscreen and avoid bright sunlight F. Avoid breathing fumes of paints, stains, and stripping compounds

A. Read labels of all liquid medications D. void alcohol-based products F. Avoid breathing fumes of paints, stains, and stripping compounds

Which guidelines should direct nursing care when deescalating an angry patient? Select All that Apply. A. Recognize the patient's need for increased personal space B. Identify the trigger for the anger C. Agree to demands as long as they won't result in harm to anyone D. Behave calmly and respectfully E. Intervene as quickly as possible

A. Recognize the patient's need for increased personal space B. Identify the trigger for the anger D. Behave calmly and respectfully E. Intervene as quickly as possible

A patient diagnosed with schizophrenia has received fluphenazine decanoate twice a month for 3 years. The clinic nurse notes that the patient grimaces and constantly smacks both lips. The patient's neck and shoulders twist in a slow, snakelike motion. Which problem would the nurse suspect? A. Tardive dyskinesia B. Pseudoparkinsonism C. Anticholinergic effects D. Tourette's syndrome

A. Tardive dyskinesia

A patient with schizophrenia tells their caseworker "I started doing good and I figured out that I was cured. I didn't need those drugs and their side effects anymore". What term applies to the inability of these patients to recognize their illness? Relapse Anosognosia Nonadherence Residual symptom

Anosognosia Anosognosia is a symptomatic inability to recognize the illness as an illness. Relapse is the cycle of getting better only to fall into the cycle of severe SMI symptoms and behaviors again. Sometimes this is due in part to nonadherence: noncooperation with one's treatment regimen.

The nurse assesses a patient diagnosed with schizophrenia. Which assessment finding would the nurse regard as a negative symptom of schizophrenia? A. Auditory hallucinations B. Poor personal hygiene C. Psychomotor agitation D. Delusions of grandeur

B. Poor personal hygiene

Indicate whether the following manifestations of a stroke are more likely to occur with right brain damage (R) or left brain damage (L) Answer CHoices: a. Right b. Left Aphasia Impaired judgment Quick, impulsive behavior Inability to remember words Left homonymous hemianopsia Neglect of the left side of the body Hemiplegia of the right side of the body

Aphasia: Left Impaired Judgement: Right Quick, impulsive behavior: Right Inability to Remember Words: Left Left homonymous hemianopsia: Right Neglect of the left side of the body: Right Hemiplegia of the right side of the body: Left

Word salad

Associative looseness using jumble of words that are meaningless to the listener

A patient with paranoia and becomes agitated and threatens to assault another staff person. He tells you, "You're the only one I think I can trust. But can I? Are you going to take their side?" Select the best initial intervention for Mr. Oswald at this point. A. Say, "If you do not calm down, seclusion will be needed." B. Address him with simple directions and a calming voice. C. Help him focus by rubbing his shoulders. D. Offer him a dose of antipsychotic medication.

B. Address him with simple directions and a calming voice. A calming voice and simple, nonemotional directions can help de-escalate the patient's anxiety. This is an initial intervention, so do not threaten him with seclusion or resort to antipsychotics. Rubbing his shoulders is inappropriate and may contribute to anxiety, not calm.

The nurse would be most concerned when the patient's breast examination reveals which finding? A. A large, tender, moveable mass in the upper inner quadrant B. An immobile, hard, nontender lesion in the upper outer quadrant C. A 2- to 3-cm, firm, defined, mobile mass in the lower outer quadrant D. A painful, moveable mass with reddened skin in the upper outer quadrant

B. An immobile, hard, nontender lesion in the upper outer quadrant On palpation, malignant lesions are characteristically hard, irregularly shaped, poorly delineated, nontender, and nonmobile and the most common site is the upper outer quadrant of the breast. Fibrocystic lesions are usually large, tender, moveable masses found throughout the breast tissue. A fibroadenoma is firm, defined, and mobile. A painful, immobile mass under a reddened area of skin is most typical of a local abscess

Which patient behavior is a criterion for mechanical restraint? A. Screaming profanities B. Assaulting a staff person C. Refusing a medication dose D. Throwing a pillow at another patient

B. Assaulting a staff person Indications for the use of mechanical restraint include protecting the patient from self-harm and preventing the patient from assaulting others.

5. Which statement(s) accurately describe(s) mild cognitive impairment (select all that apply)? A. Always progresses to AD B. Caused by variety of factors and may progress to AD C. Should be aggressively treated with acetylcholinesterase drugs D. Caused by vascular infarcts that, if treated, will delay progression to AD E. Patient is usually not aware that there is a problem with his or her memory

B. Caused by variety of factors and may progress to AD

The nurse is counseling a group of individuals over the age of 50 with average risk for cancer about screening tests for cancer. Which screening recommendation should be performed to screen for colorectal cancer? A. Barium enema every year B. Colonoscopy every 10 years C. Fecal occult blood every 5 years D. Annual prostate-specific antigen (PSA) and digital rectal exam

B. Colonoscopy every 10 years

A person has recently abused morphine. The person's pupils would most likely be A. Dilated B. Constricted C. Symmetrical D. Unresponsive to light

B. Constricted

An adult was brought to the emergency room. The patient's sensorium alternates between clouded and clear, and the patient becomes agitated both physically and verbally when approached. The patient's roommate states, "The patient was fine after getting up this morning but started talking crazy about 3 hours ago." The patient's cognitive impairment is most consistent with: A. Early-onset Alzheimer disease B. Delirium C. Sundown syndrome D. Dementia

B. Delirium

Which assessment findings are likely for an individual who recently injected heroin? A. Anxiety, restlessness, paranoid delusions B. Drowsiness, constricted pupils, slurred speech C. Muscle aching, dilated pupils, tachycardia D. Heightened sexuality, insomnia, euphoria

B. Drowsiness, constricted pupils, slurred speech

Which factor differentiates hypertensive emergency from hypertensive urgency? A. Degree of blood pressure (BP) elevation B. Presence of target end-organ involvement C. Presence of epistaxis D. Presence of headache

B. Presence of target end-organ involvement The presence of target organ involvement is associated with hypertensive emergency. The target organs of hypertension are the heart, brain, kidney, and retina.

Which assessment finding for a patient who is receiving spironolactone to treat stage 2 hypertension is most important to report to the health care provider? A. Serum potassium level of 3.0 mEq/L B. Serum potassium level of 5.5 mEq/L C. Orthostatic systolic BP decrease of 12 mm Hg D. Most recent blood pressure (BP) reading of 168/94 mm Hg

B. Serum potassium level of 5.5 mEq/L Spiranolactone (Aldactone) is a diuretic acting on the distal renal tubules Effect on preload and afterload Potassium-sparing Orthostatic drop in SBP > 20 usually requires notifying the provider Assess reasons for no change in blood pressure before reaching out to the provider

Magical thinking

Believing that thoughts or actions affect others' consequences

Sentinel lymph node biopsy (SLNB) is planned for a patient undergoing a modified radical mastectomy for breast cancer. What does the nurse teach the patient and her family about the purpose of this specific procedure? A. SLNB provides metastatic lymph nodes to test for responsiveness to chemotherapy B. If one sentinel lymph node is positive for malignant cells, all of the sentinel lymph nodes will be removed C. A radioisotope indicates which lymph nodes are most likely to have metastasis, and all of those nodes are removed D. A complete axillary lymph node dissection (ALND) will be done if sentinel lymph nodes are negative for malignant cells.

C. A radioisotope indicates which lymph nodes are most likely to have metastasis, and all of those nodes are removed In a sentinel lymph node biopsy (SLNB) radioisotopes or dye identify lymph nodes that drain from the tumor site, and they are removed. Those nodes are examined for malignant cells. If any of the nodes have malignant cells, the next step is a complete axillary lymph node dissection (ALND). If the sentinel nodes are negative, no additional lymph nodes are removed.

Which of the following are systemic effects of sublingual nitroglycerin given for acute angina (select all that apply)? A. Increased contractility B. Increased myocardial oxygen demand C. Analgesia D. Decreased preload D. Decreased afterload

C. Analgesia D. Decreased preload D. Decreased afterload

Which diagnostic test will be most useful to the nurse in determining whether a patient admitted with acute shortness of breath has heart failure? A. Serum troponin B. Arterial blood gases C. B-type natriuretic peptide D. 12-lead electrocardiogram

C. B-type natriuretic peptide

A patient has just been diagnosed with hypertension and started on captopril. Which information is most important to include when teaching the patient about this drug? A. Include high-potassium foods such as bananas in the diet B. Increase fluid intake if dryness of the mouth is a problem C. Change position slowly to help prevent dizziness and falls D. Check blood pressure in both arms before taking the drug

C. Change position slowly to help prevent dizziness and falls

A patient with schizophrenia, disorganized type, approaches the nurse and says "It's beat, it's eat. No room for doom." How will the nurse correctly assess this verbalization? A. Neologisms B. Ideas of reference C. Clanging D. Associative looseness

C. Clanging

Shortly before treatment, after crying and begging him to get help, the patient's girlfriend stayed home from a planned night out with her friends to pour all the alcohol in his apartment down the drain. What type of behavior is evident? A. Enabling B. Tolerance C. Codependence D. Use of defense mechanisms

C. Codependence

2. Vascular dementia is associated with A. Transient ischemic attacks. B. Bacterial or viral infection of neuronal tissue. C. Cognitive changes secondary to cerebral ischemia. D. Abrupt changes in cognitive function that are irreversible

C. Cognitive changes secondary to cerebral ischemia.

Profound bradycardia and hypotension are clinical manifestations most likely to be associated with which shock state? A. Anaphylactic B. Cardiogenic C. Neurogenic D. Septic

C. Neurogenic

4. Dementia with Lewy bodies (DLB) is characterized by A. Remissions and exacerbations over many years. B. Memory impairment, muscle jerks, and blindness. C. Parkinsonian symptoms, including muscle rigidity. D. Increased intracranial pressure secondary to decreased CSF drainage.

C. Parkinsonian symptoms, including muscle rigidity.

A child is being treated in the ED for injuries his father claims happened while skateboarding. The physician and nurse suspect possible child abuse. When alone with the child, he agrees to tell the nurse what happened, "as long as you keep it a secret." What should the nurse do? A. Agree to keep the child's secret to build trust. B. Ask the child's parents to explain what happened. C. Report the suspected abuse to child care services. D Have the child talk to another victim of child abuse.

C. Report the suspected abuse to child care services.

Clinical indications of left ventricular failure (LVF) include: A. jugular venous distention, hepatomegaly, and peripheral edema B. jugular venous distention, muffled heart sounds, and hypotension C. S3 at the apex, dyspnea, and crackles D. dyspnea, chest pain, and S3 at the lower sternum

C. S3 at the apex, dyspnea, and crackles The classic clinical indications of LVF are S3 at the apex, crackles over lung bases, and dyspnea. Option a describes right ventricular failure, option b describes cardiac tamponade, and option d describes pulmonary embolism. In differentiating left from right ventricular failure, remember the following: In left ventricular failure, the problem is in the system, and the symptoms are in the lungs. In right ventricular failure, the problem is in the lungs, and the symptoms are in the system.

A patient suspected of having a Non ST-segment elevation myocardial infarction (N-STEMI) has a stat troponin level drawn which yields a negative result. This finding indicates: A. No further troponin studies are required B. The physician should order a creatine kinase level since troponins have a lower specificity for cardiac muscle ischemia C. Serial troponins should be ordered q6h x 3 D. A stat repeat troponin should be drawn now to ensure the first result is not a false negative result

C. Serial troponins should be ordered q6h x 3

Child abuse perpetrators are most often A. Extended family members. B. Child care workers. C. The child's parents. D. Strangers.

C. The child's parents.

A patient is admitted with subarachnoid hemorrhage. In order to maximize venous return and reduce intracranial pressure (ICP), the patient should be positioned: A. Flat with a pillow B. Prone with the head turned to the side C. With the head of the bed elevated 30 degrees D. In high Fowler's with the knee gatch raised

C. With the head of the bed elevated 30 degrees Most patients with neurologic conditions are positioned with the HOB elevated 30 degrees. Care must be taken to avoid any compression of the jugular veins. Both options a and b would likely reduce venous drainage from the head. High Fowler's position would likely increase the intraabdominal pressure and, resultantly, the intracranial pressure.

A patient is unconscious after falling from a roof. The physician prepares to evaluate the oculocephalic reflex. What should be assessed before the oculocephalic reflex is evaluated? Cervical spine for injury Blood pressure for widened pulse pressure Motor strength for weakness Gag reflex for absence

Cervical spine for injury

A patient has dysphagia secondary to having a thrombotic stroke. Before allowing him to eat, which action should you take first? Check the patient's gag reflex. Request a soft diet with no liquids. Place the patient in high-Fowler's position. Test the patient's ability to swallow with a small amount of water.

Check the patient's gag reflex.

Drugs for Alzheimer's Disease

Cholinesterase inhibitors (slow the destruction of acetylcholine): - Tacrine (Cognex) - Donepezil (Aricept) - Galantamine (Razadyne) - Rivastigmine (Exelon) Glutamate-blocking agent: - Memantine (Namenda, Namenda XR)

During assessment, a patient remains on topic but is verbose and takes an excessive amount of time to make their point. Which speech disturbance are they exhibiting? Pressured speech Circumstantiality Flight of ideas Tangentiality

Circumstantiality

A patient is admitted with urinary tract sepsis and septic shock. Within 24 hours, hematuria, hemoptysis, petechiae and purpura, and oozing from IV sites and wounds are noted. Disseminated intravascular coagulation (DIC) is suspected. Bleeding in DIC results from: Platelet malfunction and malformation Consumption of clotting factors Hereditary absence of clotting factors Interference in the clotting pathways by toxins

Consumption of clotting factors DIC frequently is called a consumptive coagulopathy. Stimulation of the clotting cascade and formation of microclots cause the consumption of clotting factors and platelets. Stimulation of the fibrinolytic process causes formation of fibrin degradation products (FDPs) from clots. These FDPs also have an anticoagulant effect. DIC is always secondary and never hereditary or primary, so eliminate option c. DIC may be caused by septic shock and toxins but also may be caused by factors totally unrelated to toxins, so eliminate option d. Platelet numbers are decreased rather than being malformed or malfunctioning, so eliminate option a. Excessive clotting (phase 1) uses up clotting factors, causing bleeding (phase 2) and stimulation of the fibrinolytic system (phase 3). Choose option b.

A patient diagnosed with a pulmonary embolism asks the nurse why he is being started on heparin. What is the nurse's best response? A. "The heparin will thin your blood so you will be able to breathe better." B. "The heparin will dissolve any other blood clots you have." C. "It will stop any blood clots from going to your lungs." D. "Heparin will prevent any new blood clots from developing."

D. "Heparin will prevent any new blood clots from developing."

A nurse caring for a patient with Alzheimer disease can anticipate that the family will most likely need information about: A. Antimetabolites B. Benzodiazepines C. Immunosuppressants D. Acetylcholinesterase inhibitors

D. Acetylcholinesterase inhibitors Rationale: Acetylcholinesterase inhibitors are often prescribed to treat Alzheimer disease. These drugs allow greater concentration of acetylcholine in the brain, thereby improving cognitive function.

A patient becomes increasingly agitated, and when you come back from break, you find out he wants to talk to you alone. Your response is to A. Respect his privacy and see him alone. B. Do not speak to him in private; it's time for the team to confront him (calmly) as a group. C. Ask for him to be put in restraints first or take security staff with you. D. Go speak to him in a nonconfrontational way, but ensure that there are other staff nearby for backup.

D. Go speak to him in a nonconfrontational way, but ensure that there are other staff nearby for backup. Ensure that there is enough staff for backup. Only one person should talk to the patient, but staff need to maintain an unobtrusive presence in case the situation escalates. Avoid confrontation with the patient, either through verbal means or through a "show of force" with security guards. Verbal confrontation and discussion of the incident must occur when the patient is calm. A show of force by security guards may serve to escalate the patient's behavior; therefore, security personnel are better kept in the background until they are needed to assist.

3. The clinical diagnosis of dementia is based on A. CT or MRS. B. Brain biopsy. C. Electroencephalogram. D. Patient history and cognitive assessment.

D. Patient history and cognitive assessment.

A patient on the cardiac critical care unit had a myocardial infarction 2 days ago. The nurse notes the following vital signs: HR 115, BP 148/97, SpO2 89% on 6L NC, RR 30, temperature 38.0. Which vital sign is an expected finding following a myocardial infarction and does not require follow-up with the provider? A. Heart rate B. Blood pressure C. Pulse oximetry D. Temperature

D. Temperature The temperature may increase to 100.4° F (38° C) within the first 24-48 hours following an MI The temperature elevation may last for as long as 4-5 days. This increase in temperature is due to a systemic inflammatory process caused by the death of heart cells.

Downs Sydrome

Genetic/chromosomal condition

A patient who was involved in a motor vehicle crash presents to the emergency department with the following arterial blood gas results: pH 7.48, PaO2 85 mm Hg, PaCO2 32 mm Hg, and HCO3 25 mEq/L. What is the priority nursing intervention? A. Administer 4 mg morphine and 1 mg lorazepam (Ativan) IV push immediately B. Have the patient slowly breathe in and out of a paper bag C. Place the patient on oxygen at 6L NC D. Prepare for immediate intubation

Have the patient slowly breathe in and out of a paper bag

A nurse is caring for a patient admitted for an ischemic stroke to the right middle cerebral artery. The nurse would expect the patient to exhibit which deficit specific to a right-sided stroke? Impulsive behavior Slow to perform tasks Fearfulness and anxiety Difficulty in responding to verbal cues

Impulsive behavior Impulsive behavior is commonly seen in right-sided strokes whereas people with left-sided strokes tend to be more cautious and slow to perform tasks. People with left-sided strokes tend to exhibit more fearfulness and anxiety and respond better to non-verbal cues (rather than verbal cues).

Tangentiality

Including unnecessary and often tedious details in conversation, but eventually reaching the point

Circumstantiality

Including unnecessary and often tedious details in conversation, but eventually reaching the point.

FASD

Leading cause of intellectual disability in U.S. Caused by alcohol abuse during pregnancy (35% risk) Social and behavioral problems (often misdiagnosed; diagnosed late) Many system-level barriers to receiving needed support

Which aspect of assessment is the most important in a patient with a platelet count of 10,000/mm3? Skin assessment for petichiae and echymosis Level of consciousness Blood pressure Urine output

Level of consciousness Patients with platelet counts this low are at great risk of spontaneous intracerebral hemorrhage. This would be manifested by a change in level of consciousness, headache, pupillary changes, and/or focal signs such as hemiparesis, hemiplegia, or aphasia.

Flight of ideas

Moving rapidly from one thought to the next, often making it difficult for others to follow the conversation

A patient was admitted to the critical care unit after an automobile collision in which their forehead struck the windshield. In this situation, the contrecoup injury would occur in the: Temporal region Parietal region Frontal region Occipital region

Occipital region Consider the rule of trauma that everything moves until it stops. The brain moves forward with impact causing the coup injury to the frontal lobe and then moves backward with impact causing the contrecoup injury to the occipital lobe.

Which laboratory values would differentiate diabetic ketoacidosis (DKA) from a hyperosmolar hyperglycemic state (HHS)? Serum glucose of 600 mg/dL Serum potassium of 4 mEq/L Positive serum ketones Serum osmolality of 320 mOsm/L

Positive serum ketones In DKA there is an absolute insulin deficiency that causes glycogenolysis and gluconeogenesis. The gluconeogenesis causes the incomplete breakdown of free fatty acids, which results in ketones in the blood and urine. In HHS, there is a relative insulin deficiency that causes glycogenolysis but does not cause gluconeogenesis. Therefore tests for ketones are positive in DKA but typically negative in HHS.

The patient becomes anxious and says, "There are worms under my skin eating the hair follicles." How would you classify this assessment finding? Positive symptom Negative symptom Cognitive symptom Depressive symptom

Positive symptoms

A patient has disorganized thoughts, auditory hallucinations, and insists on being addressed as "Mr. President." Which of the following is the patient's most likely diagnosis? Psychotic episode Depression Mania Anxiety

Psychotic episode

Which are clinical findings in a person with an acute lower extremity VTE (select all that apply)? a. Pallor and coolness of foot and calf b. Mild to moderate calf pain and tenderness c. Grossly diminished or absent pedal pulses d. Unilateral edema and induration of the thigh e. Palpable cord along a superficial varicose vein

b. Mild to moderate calf pain and tenderness

While caring for a patient who is at the nadir of chemotherapy, the nurse establishes the highest priority for nursing actions related to: Diarrhea Grieving Risk for infection Inadequate nutritional intake

Risk for infection The nadir is the point of the lowest blood counts after chemotherapy is started, and it is the time when the patient is most at risk for infection. Because infection is the most common cause of morbidity and death in cancer patients, identification of risk and interventions to protect the patient are of the highest priority. The other problems will be treated, but they are not the priority.

Cognitive symptoms

Subtle or obvious impairment in memory, attention, thinking (e.g., disorganized or irrational thoughts), judgment or problem solving

The critical care nurse is caring for a type 1 diabetic patient admitted for diabetic ketoacidosis. The patient is receiving 0.9% NS at 250 mL/hr and is on an insulin drip at a rate of 12 units per hour. The nurse reviews the most recent basic metabolic panel results which include: Serum glucose: 224; Anion gap: 23; and CO2: 8. The nurse's next intervention should be to: Turn off the insulin drip Administer 25mL 50% Dextrose (D50) IV Increase the IV insulin drip to 14 units/hour Switch the IV fluid solution to one that contains 5% dextrose (e.g. D5 0.9% NS)

Switch the IV fluid solution to one that contains 5% dextrose (e.g. D5 0.9% NS)

Affective symptoms

Symptoms involving emotions and their expression.

A patient is admitted with ischemic stroke. The pupils are unequal with one pupil fully dilated. Which of the following statements regarding this finding is most accurate? The stroke is located in the pons This is a normal finding in a small percentage of the population This is a side effect of narcotics administration This is an indication of increased intracranial pressure

This is an indication of increased intracranial pressure

Negative symptoms

The absence of something that should be present. Negative symptoms include the ability to enjoy activities, social comfort, or goal-directed behavior.

Positive symptoms

The presence of something that should not be present. Positive symptoms include hallucinations, delusions, paranoia, or disorganized or bizarre thought, behavior or speech. These symptoms usually appear early.

Thought insertion

The uncomfortable belief that someone else has inserted thoughts into your brain

The following are associated with right-sided heart failure except: a. Frothy sputum production b. Jugular venous distention c. Increased peripheral edema d. Cor pulmonale

a. Frothy sputum production

A patient admitted for unstable angina on the telemetry floor of the hospital complains of sharp chest pain radiating to his jaw and down his left shoulder. His VS are HR 104, BP 145/93, SpO2 88% on room air. He is pale and diaphoretic. Prioritize the following interventions in order from most important to least important: 1. Draw a stat troponin level 2. Perform a stat electrocardiogram (EKG) 3. If indicated by the EKG results, give nitroglycerin (NG) 0.04 mg SL q 5 minutes x 3 doses 4. Place the patient on 2LNC and titrate O2 to SpO2 > 92% a. 1, 4, 2, 3 b. 1, 2, 3, 4 c. 4, 2, 3, 1 d. 4, 1, 2, 3

c. 4, 2, 3, 1 Ischemia indicates a lack of oxygen to meet myocardial demand ECG will help determine if infarction is occurring and a general idea of which arteries are occluded NG isn't indicated for every MI Troponin levels are important but can be drawn last

A patient admitted to the critical care unit after a motor vehicle collision was unconscious at the scene of the accident. He is now alert and oriented. Skull films show a linear fracture of the right temporal bone. He is at significant risk for: a. Scalp laceration b. Subdural hematoma c. Epidural hematoma d. Intracerebral hematoma

c. Epidural hematoma Linear fractures of the temporal bone frequently disrupt the middle meningeal artery and cause epidural hematoma. Patients with an epidural hematoma classically present with a short period of unconsciousness followed by a lucid interval and then rapid deterioration. An epidural hematoma is usually caused by arterial bleeding.

Which patient is most at risk for developing delirium? a. A 50-yr-old woman with cholecystitis b. A 19-yr-old man with a fractured femur c. A 42-yr-old woman having an elective hysterectomy d. A 78-yr-old man admitted to the medical unit with complications related to heart failure

d. A 78-yr-old man admitted to the medical unit with complications related to heart failure

A patient is diagnosed with a thrombotic stroke. Over the next 72 hours, you plan care with the knowledge that he is ready for aggressive rehabilitation. will show gradual improvement of the initial neurologic deficits. may show signs of deteriorating neurologic function as cerebral edema increases. should not be turned or exercised to prevent extension of the thrombus and increased neurologic deficits.

may show signs of deteriorating neurologic function as cerebral edema increases.

When planning care for a patient at risk for pulmonary embolism, the nurse prioritizes a. maintaining the patient on bed rest. b. using sequential compression devices. c. encouraging the patient to cough and deep breathe. d. teaching the patient how to use the incentive spirometer.

using sequential compression devices

Bladder training in a male patient who has urinary incontinence after a stroke includes a. limiting fluid intake. b. keeping a urinal in place at all times. c. assisting the patient to stand to void. d. catheterizing the patient every 4 hours.

c. assisting the patient to stand to void.

The nurse teaches a client with diabetes mellitus about differentiating between hypoglycemia and ketoacidosis. The client demonstrates an understanding of the teaching by stating that a form of glucose should be taken if which symptom or symptoms develop? Select all that apply. 1. Polyuria 2. Shakiness 3. Palpitations 4. Blurred vision 5. Lightheadedness 6. Fruity breath odor

2. Shakiness 3. Palpitations 5. Lightheadedness

A client with a diagnosis of diabetic ketoacidosis (DKA) is being treated in the emergency department. Which findings support this diagnosis? Select all that apply. 1. Increase in pH 2. Comatose state 3. Deep, rapid breathing 4. Decreased urine output 5. Elevated blood glucose level

2. Comatose state 3. Deep, rapid breathing 5. Elevated blood glucose level

The nurse is caring for a client admitted to the emergency department with diabetic ketoacidosis (DKA). In the acute phase, the nurse plans for which priority intervention? 1. Correct the acidosis. 2. Administer 5% dextrose intravenously. 3. Apply a monitor for an electrocardiogram. 4. Administer short-duration insulin intravenously.

4. Administer short-duration insulin intravenously.

As one of your clinical assignments, you are assisting an RN with health screening at a health fair. Which individual is at greatest risk for experiencing a stroke? A 46-year-old white female with hypertension and oral contraceptive use for 10 years A 58-year-old white male salesman who has a total cholesterol level of 285 mg/dl A 42-year-old African American female with diabetes mellitus who has smoked for 30 years A 62-year-old African American male with hypertension who is 35 pounds overweight

A 62-year-old African American male with hypertension who is 35 pounds overweight

What is a compensatory mechanism for metabolic alkalosis? A. Decreased respiratory rate and depth to retain CO2 and kidney excretion of bicarbonate B. Kidney conservation of bicarbonate and excretion of hydrogen ions C. Deep, rapid respirations (Kussmaul respirations) to increase CO2 excretion D. Shifting of bicarbonate into cells in exchange for chloride

A. Decreased respiratory rate and depth to retain CO2 and kidney excretion of bicarbonate

The surgery area calls the transfer report for a 68-year-old, postmenopausal, female patient who smokes and takes hormone therapy. She is returning to the clinical unit after a lengthy hip replacement surgery. Which factors present in this patient increase her risk for developing venous thromboembolism (VTE) related to Virchow's triad (select all that apply)? A. Estrogen therapy B. IV therapy C. Smoking D. Dehydration E. Prolonged immobilization F. Orthopedic surgery

A. Estrogen therapy B. IV therapy C. Smoking E. Prolonged immobilization F. Orthopedic surgery

A patient who is lethargic and exhibits deep, rapid respirations has the following arterial blood gas (ABG) results: pH 7.32, PaO2 88 mm Hg, PaCO2 37 mm Hg, and HCO3 16 mEq/L. How should the nurse interpret these results? A. Metabolic acidosis B. Metabolic alkalosis C. Respiratory acidosis D. Respiratory alkalosis

A. Metabolic acidosis

A newly admitted patient diagnosed with a right-sided brain stroke has a nursing diagnosis of disturbed visual sensory perception related to homonymous hemianopsia. Early in the care of the patient, what should the nurse do? A. Place objects on the right side within the patient's field of vision. B. Approach the patient from the left side to encourage the patient to turn the head. C. Place objects on the patient's left side to assess the patient's ability to compensate. D. Patch the affected eye to encourage the patient to turn the head to scan the environment.

A. Place objects on the right side within the patient's field of vision.

A 27-yr-old patient admitted with diabetic ketoacidosis (DKA) has a serum glucose level of 732 mg/dL and serum potassium level of 3.1 mEq/L. Which action prescribed by the health care provider should the nurse take first? A. Place the patient on a cardiac monitor B. Administer IV potassium chloride C. Ask the patient about home insulin doses D. Start an insulin infusion at 0.1 units/kg/hr

A. Place the patient on a cardiac monitor

A patient is admitted with a headache, fever, and general malaise. The HCP has asked that the patient be prepared for a lumbar puncture. What is a priority nursing action to avoid complications? a. Ensure that CT scan is performed prior to lumbar puncture. b. Assess laboratory results for changes in the white cell count. c. Provide acetaminophen for the headache and fever before the procedure. d. Administer antibiotics before the procedure to treat the potential meningitis.

a. Ensure that CT scan is performed prior to lumbar puncture.

A patient with Type 1 Diabetes arrives in the emergency department having run out of insulin for his implanted pump and is diagnosed with diabetic ketoacidosis. Which of the following is the priority nursing intervention? Administer 20 units insulin glargine (Lantus) subcutaneously Begin aggressive fluid resuscitation (e.g. 0.9% NS at 1L per hour x 5L total) Begin IV insulin drip at rate of 0.1 units/kg/hour Ensure patient refills the insulin for his implanted pump

Begin aggressive fluid resuscitation (e.g. 0.9% NS at 1L per hour x 5L total)

The patient comes to the HCP office with pain, edema, and warm skin on her lower left leg. What test should the nurse expect to be ordered first? A. Complete blood count (CBC) B. Magnetic resonance imaging (MRI) C. Duplex ultrasound D. Computed venography (phlebogram)

C. Duplex ultrasound

A carotid endarterectomy is being considered as treatment for a patient who has had several TIAs. What should the nurse explain to the patient about this surgery? A. It involves intracranial surgery to join a superficial extracranial artery to an intracranial artery B. It is used to restore blood circulation to the brain following an obstruction of a cerebral artery. C. It involves removing an atherosclerotic plaque in the carotid artery to prevent an impending stroke. D. It is used to open a stenosis in a carotid artery with a balloon and stent to restore cerebral circulation.

C. It involves removing an atherosclerotic plaque in the carotid artery to prevent an impending stroke.

To help prevent embolization of a thrombus in a patient with acute VTE and severe edema and limb pain, what should the nurse teach the patient to do first? A. Ambulate around the bed three to four times a day. B. Dangle the feet over the edge of the bed q2-3hr. C. Keep the affected leg elevated above the level of the heart. D. Maintain bed rest until edema is relieved and anticoagulation is established.

D. Maintain bed rest until edema is relieved and anticoagulation is established.

Which assessment finding for a patient diagnosed with a right calf venous thromboembolism (VTE) requires immediate action by the nurse? A. Temperature of 100.4 degrees F (38 degrees C) B. Erythema of right lower leg C. Complaint of right calf pain D. New onset shortness of breath

D. New onset shortness of breath

What is the neurologic diagnostic test that has the highest risk of complications and requires frequent monitoring of neurologic and vital signs following the procedure? a. Electromyelogram b. Cerebral angiography c. Electroencephalogram d. Transcranial Doppler sonography

b. Cerebral angiography

Intravenous heparin therapy is prescribed for a client. While implementing this prescription, the nurse ensures that which medication is available on the nursing unit? 1. Vitamin K 2. Protamine sulfate 3. Potassium chloride 4. Aminocaproic acid

2. Protamine sulfate The antidote to heparin is protamine sulfate; it should be readily available for use if excessive bleeding or hemorrhage should occur. Vitamin K is an antidote for warfarin sodium. Potassium chloride is administered for a potassium deficit. Aminocaproic acid is the antidote for thrombolytic therapy.

Psychosis/Schizophrenia Assessment

- Assess for hallucinations Do you recognize the voices? Do you believe the voices are real? What are they telling you to do? Do you plan to follow the command? - Assess for delusions - Assess for suicide risk - Assess ability to ensure personal safety and health - Assess prescribed meds - Assess symptoms' impact on functioning - Assess family knowledge

First Generation (typicals)

- Dopamine antagonists (D2 receptor antagonists) - Target positive symptoms of schizophrenia Advantage - Less expensive than second generation Disadvantages - Extrapyramidal side effects (EPS) - Anticholinergic (ACh) side effects - Tardive dyskinesia - Weight gain, sexual dysfunction, endocrine disturbances

Characteristics of Vulnerable Persons: Older adults

- Poor mental or physical health - Dependent on perpetrator - Female, older than 75 years, white, living with a relative - Elderly father cared for by a daughter he abused as a child - Elderly woman cared for by a husband who has abused her in the past

Second Generation (atypicals)

- Serotonin (5-HT2A receptor) and dopamine (D2 receptor) antagonists - Treat both positive and negative symptoms - Minimal to no EPS or tardive dyskinesia - Disadvantage—tendency to cause significant weight gain; risk of metabolic syndrome

Characteristics of Vulnerable Persons: Children

- Younger than 3 years - Perceived as different - Remind parents of someone they do not like - Product of an unwanted pregnancy - Interference with emotional bonding between parent and child

The nurse determines that the wife of an alcoholic client is benefiting from attending an Al-Anon group if the nurse hears the wife make which statement? 1. "I no longer feel that I deserve the beatings my husband inflicts on me." 2. "My attendance at the meetings has helped me to see that I provoke my husband's violence." 3. "I enjoy attending the meetings because they get me out of the house and away from my husband." 4. "I can tolerate my husband's destructive behaviors now that I know they are common among alcoholics.

1. "I no longer feel that I deserve the beatings my husband inflicts on me."

A client with a clot in the right atrium is receiving a heparin sodium infusion at 1000 units/hour and warfarin sodium 7.5 mg at 5:00 p.m. daily. The morning laboratory results are as follows: activated partial thromboplastin time (aPTT), 32 seconds; international normalized ratio (INR), 1.3. The nurse should take which action based on the client's laboratory results? 1. Collaborate with the health care provider (HCP) to discontinue the heparin infusion and administer the warfarin sodium as prescribed. 2. Collaborate with the HCP to obtain a prescription to increase the heparin infusion and administer the warfarin sodium as prescribed. 3. Collaborate with the HCP to withhold the warfarin sodium since the client is receiving a heparin infusion and the aPTT is within the therapeutic range. 4. Collaborate with the HCP to continue the heparin infusion at the same rate and to discuss use of dabigatran etexilate in place of warfarin sodium.

2. Collaborate with the HCP to obtain a prescription to increase the heparin infusion and administer the warfarin sodium as prescribed. When a client is receiving warfarin for clot prevention due to atrial fibrillation, an INR of 2 to 3 is appropriate for most clients. Until the INR has achieved a therapeutic range, the client should be maintained on a continuous heparin infusion with the aPTT ranging between 60 and 80 seconds. Therefore, the nurse should collaborate with the HCP to obtain a prescription to increase the heparin infusion and to administer the warfarin as prescribed.

The nurse is assessing the motor and sensory function of an unconscious client. The nurse should use which technique to test the client's peripheral response to pain? 1. Sternal rub 2. Nail bed pressure 3. Pressure on the orbital rim 4. Squeezing of the sternocleidomastoid muscle

2. Nail bed pressure

The nurse should report which assessment finding to the health care provider (HCP) before initiating thrombolytic therapy in a client with pulmonary embolism? 1. Adventitious breath sounds 2. Temperature of 99.4 °F (37.4 °C) orally 3. Blood pressure of 198/110 mm Hg 4. Respiratory rate of 28 breaths/minute

3. Blood pressure of 198/110 mm Hg Thrombolytic therapy is contraindicated in a number of preexisting conditions in which there is a risk of uncontrolled bleeding, similar to the case in anticoagulant therapy. Thrombolytic therapy also is contraindicated in severe uncontrolled hypertension because of the risk of cerebral hemorrhage. Therefore, the nurse would report the results of the blood pressure to the HCP before initiating therapy.

A client has experienced pulmonary embolism. The nurse should assess for which symptom, which is most commonly reported? 1. Hot, flushed feeling 2. Sudden chills and fever 3. Chest pain that occurs suddenly 4. Dyspnea when deep breaths are taken

3. Chest pain that occurs suddenly The most common initial symptom in pulmonary embolism is chest pain that is sudden in onset. The next most commonly reported symptom is dyspnea, which is accompanied by an increased respiratory rate. Other typical symptoms of pulmonary embolism include apprehension and restlessness, tachycardia, cough, and cyanosis.

A client is admitted to a hospital with a diagnosis of diabetic ketoacidosis (DKA). The initial blood glucose level is 950 mg/dL (54.2 mmol/L). A continuous intravenous (IV) infusion of short-acting insulin is initiated, along with IV rehydration with normal saline. The serum glucose level is now decreased to 240 mg/dL (13.7 mmol/L). The nurse would next prepare to administer which medication? 1. An ampule of 50% dextrose 2. NPH insulin subcutaneously 3. IV fluids containing dextrose 4. Phenytoin for the prevention of seizures

3. IV fluids containing dextrose

A client is brought to the emergency department in an unresponsive state, and a diagnosis of hyperosmolar hyperglycemic syndrome is made. The nurse would immediately prepare to initiate which anticipated health care provider's prescription? 1. Endotracheal intubation 2. 100 units of NPH insulin 3. Intravenous infusion of normal saline 4. Intravenous infusion of sodium bicarbonate

3. Intravenous infusion of normal saline

A client receiving thrombolytic therapy with a continuous infusion of alteplase suddenly becomes extremely anxious and complains of itching. The nurse hears stridor and notes generalized urticaria and hypotension. Which nursing action is the priority? 1. Administer oxygen and protamine sulfate. 2. Cut the infusion rate in half and sit the client up in bed. 3. Stop the infusion and call for the Rapid Response Team (RRT). 4. Administer diphenhydramine and epinephrine and continue the infusion.

3. Stop the infusion and call for the Rapid Response Team (RRT). The client is experiencing an anaphylactic reaction. Therefore, the priority action is to stop the infusion and notify the RRT. The health care provider should be contacted once the client has been stabilized. The client may be treated with epinephrine, antihistamines, and corticosteroids as prescribed, but the infusion should not be continued.

The nurse provides instructions to a client newly diagnosed with type 1 diabetes mellitus. The nurse recognizes accurate understanding of measures to prevent diabetic ketoacidosis when the client makes which statement? 1. "I will stop taking my insulin if I'm too sick to eat." 2. "I will decrease my insulin dose during times of illness." 3. "I will adjust my insulin dose according to the level of glucose in my urine." 4. "I will notify my health care provider (HCP) if my blood glucose level is higher than 250 mg/dL (14.2 mmol/L)."

4. "I will notify my health care provider (HCP) if my blood glucose level is higher than 250 mg/dL (14.2 mmol/L)."

The nurse provides discharge instructions to a client who is taking warfarin sodium. Which statement, by the client, reflects the need for further teaching? 1. "I will avoid alcohol consumption." 2. "I will take my pills every day at the same time." 3. "I have already called my family to pick up a MedicAlert bracelet." 4. "I will take coated aspirin for my headaches because it will coat my stomach."

4. "I will take coated aspirin for my headaches because it will coat my stomach." Aspirin-containing products need to be avoided when a client is taking this medication. Alcohol consumption should be avoided by a client taking warfarin sodium. Taking the prescribed medication at the same time each day increases client compliance. The MedicAlert bracelet provides health care personnel with emergency information.

A patient initially complained of dizziness, right-sided weakness, and diplopia. These symptoms subsided within 4 hours. Which of the following statements is true? The patient had a stroke The patient had a concussion The patient had a diffuse axonal injury The patient had a transient ischemic attack

The patient had a transient ischemic attack

A 16-year-old girl with a history of type 1 diabetes mellitus (DM) is admitted to the critical care unit. Her friend states that she has had a cold for the last few days. She is now lethargic. Which of the following lab results would the nurse expect? A. Hyperglycemia, hypokalemia, acidosis, elevated serum osmolality B. Hyperglycemia, hyperkalemia, acidosis, elevated serum osmolality C. Hyperglycemia, hypernatremia, alkalosis, decreased serum osmolality D. Hypoglycemia, hyponatremia, acidosis, decreased serum osmolality

B. Hyperglycemia, hyperkalemia, acidosis, elevated serum osmolality The hyperglycemia is caused by insulin deficiency and therefore the inability of insulin to move into the cell. This hyperglycemia causes a hypertonic diuresis, dehydration, and elevated serum osmolality. Gluconeogenesis causes the breakdown of fats and proteins for energy. This results in an increase in ketone bodies and acidosis. This acidosis causes potassium to move out of the cell and into the serum, causing hyperkalemia.

The following evidence-based techniques may be used to prevent catheter-associated urinary tract infections (CAUTI) (select all that apply): A. Provide twice daily perineal care using chlorhexidine gluconate B. Keep the collection bag below the level of the bladder at all times C. Empty the collection bag only when it is full D. Collect urine for lab specimens from the sample port using a sterile syringe E. Anchor the catheter tubing to the patient's leg or abdomen

B. Keep the collection bag below the level of the bladder at all times D. Collect urine for lab specimens from the sample port using a sterile syringe E. Anchor the catheter tubing to the patient's leg or abdomen

6. The early stage of AD is characterized by A. No noticeable change in behavior. B. Memory problems and mild confusion. C. Increased time spent sleeping or in bed. D. Incontinence, agitation, and wandering behavior.

B. Memory problems and mild confusion.

A patient presents to the emergency department in the initial stages of septic shock. As septic shock worsens, the nurse anticipates which of the following PaCO2 changes as a result of respiratory failure? A. PaCO2 is initially low and then decreases further B. PaCO2 is initially low and then increases C. PaCO2 is initially high and then decreases D. PaCO2 is initially high and then increases further

B. PaCO2 is initially low and then increases The initial response to septic shock is tachypnea and hyperventilation which decreases PaCO2 As respiratory function worsens PaCO2 will increase resulting in carbon dioxide retention in the bloodstream (hypercapnia)

After receiving change-of-shift report on a heart failure unit, which patient should the nurse assess first? A. Patient who is taking carvedilol (Coreg) and has a heart rate of 58 B. Patient who is taking digoxin and has a potassium level of 3.1 mEq/L C. Patient who is taking captopril and has a frequent nonproductive cough D. Patient who is taking isosorbide dinitrate/hydralazine (BiDil) and has a headache

B. Patient who is taking digoxin and has a potassium level of 3.1 mEq/L Side effects: Bradycardia HYPOkalemia Antidote= Digibind

A patient is admitted to the critical care unit after an acute myocardial infarction. Two days later the patient develops pericarditis. When providing pain relief for patients with pericarditis, the nurse would administer an oral nonsteroidal antiinflammatory agent and instruct the patient to do which of the following? A. Refrain from breathing deeply. B. Sit up and lean forward C. Lie flat on the left side. D. Not move

B. Sit up and lean forward The pain of pericarditis is lessened when sitting up and leaning forward. When the patient is upright and leaning forward, the heart is more vertical and free, hanging with minimal pressure on adjacent structures.

45-year-old patient shows marked cognitive impairment that has developed progressively over several months. A family member reports the patient's father had early-onset dementia. What research-based information can be given to the family in response to their concerns about the patient developing early-onset dementia? A. The risk for developing the condition is about 50% only if both parents were affected. B. The greatest risk exists for relatives of individuals diagnosed with Alzheimer disease before age 55 years. C. Added risk is present only for people with Down syndrome, so relatives without Down syndrome are essentially safe D. Results of the research on genetic predisposition and its effect on the development of early-onset dementia are still unclear.

B. The greatest risk exists for relatives of individuals diagnosed with Alzheimer disease before age 55 years. Rationale: The risk for development of Alzheimer disease (AD) is greater for relatives of people with the illness than it is for those with no family history of AD. An individual with one parent with early-onset AD has a 50% chance of developing it before the age of 55 years as well. Those offspring who do not inherit early-onset AD do not pass it on to their own children and presumably have the same risk of developing AD much later in life as does the general population. Down syndrome does appear to be a risk factor for early-onset AD

A patient admitted from the emergency department is unresponsive after a suspected heroin overdosage. In assessing his pupils, you expect to find which of the following? Bilaterally dilated pupils Unequal pupils Bilaterally constricted pupils Nystagmus

Bilaterally constricted pupils Opiate intoxication causes pupils to be bilaterally pinpoint. Opiates have a diffuse, not focal, effect on the brain, so eliminate option b. Dilated pupils are related to atropine, sympathomimetic drugs, or brainstem lesions, so eliminate option a. Nystagmus usually is associated with cerebellar problems, so eliminate option d. Choose option c.

Cognition

Bizarre behavior, disorganized thoughts, and the presence of delusions are classic signs of psychosis. The person with mania does not have hallucinations, which distinguishes it from psychosis.

Pressured speech

Urgent or intense and resists comments from others.

An older patient is admitted to the hospital with a urinary infection and possible bacterial sepsis. The family is concerned because the patient is confused and not able to carry on a conversation. Which statement by the nurse is most appropriate? A. "Depression is a common cause of confusion in older adults in the hospital." B. "It is normal for an older person to have cognitive problems while in the hospital." C. "The mental changes are most likely caused by the infection and most often reversible." D. "Drug therapy with antipsychotic agents is indicated to slow the progression of dementia."

C. "The mental changes are most likely caused by the infection and most often reversible." Rationale: Delirium, a state of temporary but acute mental confusion, is a common, life-threatening, and possibly preventable syndrome in older adults. Clinically, delirium is rarely caused by a single factor. It is often the result of the interaction of the patient's underlying condition with a precipitating event.

A student nurse and his preceptor are caring for a patient diagnosed with cellulitis to the right leg. Which statement made by the student nurse requires follow-up from the nurse preceptor? A. "Cellulitis may be caused by insect bites." B. "Cellulitis may result from previous antibiotic administration." C. "The patient's right leg will probably have edema, pallor, and pain." D. "Vancomycin may be used to treat cellulitis."

C. "The patient's right leg will probably have edema, pallor, and pain." Cellulitis is often caused by a break in the skin It may also be occur as a secondary infection The inflammation process includes warmth, redness, swelling, and localized pain Treatment may require wide-spectrum IV antibiotics such as vancomycin and linezolid

A patient diagnosed with alcohol use disorder asks, "How will Alcoholics Anonymous (AA) help me?" Select the nurse's best response. A. The goal of AA is for members to learn controlled drinking with the support of a higher power." B. "You will be assigned a sponsor who will plan your treatment program." C. "You must make a commitment to permanently abstain from alcohol and other drugs." D. "An individual is supported by peers while striving for abstinence one day at a time."

D. "An individual is supported by peers while striving for abstinence one day at a time." Rationale: Admitting to being an alcoholic, making an attempt to remain alcohol-free for a day at a time, and receiving support from peers are basic aspects of AA. The other options are incorrect.

A provider has ordered phenazopyridine (Pyridium) for an elderly patient diagnosed with a urinary tract infection. The patient calls the nurse advice line reporting her urine is orange-colored. The most appropriate response the nurse should make is: A. "This is likely a hepatotoxic side effect and you should go to the emergency room for evaluation." B. "Have you also taken Rifaximin recently?" C. "Have you been drinking 4-6 liters of water daily?" D. "This is an expected side effect of this medication."

D. "This is an expected side effect of this medication." Orange-colored urine is an expected outcome of using phenazopyridine (Pyridium) While Rifaximin may also cause orange-colored urine there is no mention of its use in the question Patients in liver failure or who are dehydrated may have amber-colored urine

During the third week of treatment, the spouse of a patient in a rehabilitation program for substance abuse says, "After this treatment program, I think everything will be all right." Which remark by the nurse will be most helpful to the spouse? A. "It is good that you are supportive of your spouse's sobriety and want to help maintain it." B. "It will be important for you to structure life to avoid as much stress as you can and provide social protection." C. "Addiction is a lifelong disease of self-destruction. You will need to observe your spouse's behavior carefully." D. "While sobriety solves some problems, new ones may emerge as one adjusts to living without drugs and alcohol."

D. "While sobriety solves some problems, new ones may emerge as one adjusts to living without drugs and alcohol."

In order to counteract the body's homeostatic response to decreased cardiac output, patients with a new diagnosis of heart failure may be prescribed which classes of medications prior to discharge from the hospital? A. Angiotensin-II receptor blockers and Calcium-channel blockers B. Beta-blockers and loop diuretics C. Angiotensin converting enzyme inhibitors and Calcium-channel blockers D. Angiotensin converting enzyme inhibitors and Beta-blockers

D. Angiotensin converting enzyme inhibitors and Beta-blockers

A nurse is assigned the care of four patients who are detoxifying from alcohol. The patient with which symptom would be the nurse's highest priority? A. Fine motor tremors B. Diaphoresis C. Diarrhea D. Hallucinations and delusions

D. Hallucinations and delusions

The daughter of a patient with early familial Alzheimer's disease (AD) asks how AD is different from forgetfulness. You describe early warning signs of AD, including A. Forgetting a colleague's name at a party B. Repeatedly misplacing car keys or a wallet C. Leaving a pot on the stove that boils dry and burns D. Having no memory of preparing a meal and forgetting to serve or eat it

D. Having no memory of preparing a meal and forgetting to serve or eat it Rationale: Memory loss that affects job skills: Frequent forgetfulness or unexplainable confusion at home or in the workplace may signal that something is wrong. This type of memory loss goes beyond forgetting an assignment, a colleague's name, a deadline, or a phone number. Difficulty performing familiar tasks: It is not abnormal for most people to become distracted and to forget something (e.g., leave something on the stove too long). People with Alzheimer's disease (AD) may cook a meal but then forget not only to serve it but also that they made it. Misplacing things: For many individuals, temporarily misplacing keys, purses, or wallets is a normal albeit frustrating event. Persons with AD may put items in inappropriate places (e.g., eating utensils in clothing drawers) but have no memory of how they got there.

An elderly patient is admitted with pneumonia. This morning he is febrile, tachycardic, tachypneic, and confused. Which of the following is the initial response of the cardiovascular system to sepsis? A. Bradycardia B. Decreased contractility C. Increased preload D. Increased cardiac output

D. Increased cardiac output Contractility and cardiac output are increased in sepsis and early septic shock. The heart rate increases. Preload and afterload decrease because of massive vasodilation.

Which statement is an accurate depiction of sexual assault? A. Rape is a sexual act B. Women are usually raped by strangers C. Rape is usually an impulsive act D. Most rapes occur in the home

D. Most rapes occur in the home

A patient is brought by EMS to a rural health clinic complaining of crushing substernal chest pain radiating to the jaw and left arm. The patient's electrocardiogram shows ST-segment elevation in leads V3 and V4; vital signs are BP 210/135, temp 37.2, RR 24, SpO2 94% on 2LNC; the patient has an 18G IV to the right antecubital vein. The closest facility offering percutaneous coronary intervention (PCI) is 250 miles away but helicopters are grounded due to a blizzard. The priority IV medication the emergency department nurse should administer at this time is: A. Alteplase IV push B. Heparin gtt C. Morphine IV push D. Nitroprusside gtt

D. Nitroprusside gtt

Loose associations in a person with schizophrenia indicate A. paranoia. B. Mood instability. C. Depersonalization. D. Poorly organized thinking

D. Poorly organized thinking

The physician gives the registered nurse an order for 325 mg Ecotrin (enteric coated aspirin) PO once for a patient diagnosed with a suspected myocardial infarction. The nurse should: A. Have the patient swallow the aspirin whole with a sip of water B. Chew the aspirin quickly and swallow to promote easier absorption C. Ignore the order if the patient reports his chest pain has improved with administration of the SL nitroglycerin D. Question the physician's order

D. Question the physician's order Enteric-coated aspirin should not be crushed/chewed because it's time-released Regular aspirin should be chewed, not taken with water Ignoring an order and starting NG is out of the nurse's scope of practice

Acute kidney injury in a patient with multiple organ dysfunction syndrome (MODS) usually is related to which of the following? A. Sepsis B. Preexisting renal insufficiency C. Rhabdomyolysis D. Renal hypoperfusion

D. Renal hypoperfusion A mean arterial pressure of less than 60 mm Hg for as short a period as 40 minutes may result in acute tubular necrosis and acute kidney injury. Prolonged hypoperfusion is the most likely cause of acute tubular necrosis in a patient with MODS. Sepsis should not be confused with septic shock Preexisting renal insufficiency and rhabdomyolysis affect kidney function but are not related to septic shock and MODS

A patient with schizophrenia refuses to take his medication because he believes he is not ill. What is the most likely explanation for this belief? A. Stigma causes the patient to refuse to admit his mental illness. B. The patient is unable to face having an illness and is in denial. C. Command hallucinations are instructing him to deny the illness. D. The illness itself is preventing the patient from realizing he is ill.

D. The illness itself is preventing the patient from realizing he is ill.

Upon auscultation of a pediatric patient with known respiratory syncytial virus (RSV), the registered nurse is most likely to document which adventitious breath sound? A. Crackles B. Rhonchi (coarse) C. Stridor D. Wheezes

D. Wheezes

1. Dementia is defined as a A. syndrome that results only in memory loss. B. disease associated with abrupt changes in behavior. C. disease that is always due to reduced blood flow to the brain. D. syndrome characterized by cognitive dysfunction and loss of memory.

D. syndrome characterized by cognitive dysfunction and loss of memory.

Autism Spectrum Disorder

Deficits in social relatedness and relationships Stereotypical repetitive speech Obsessive focus on specific objects Over adherence to routines or rituals Hyper- or hypo-reactivity to sensory input Extreme resistance to change Appears in early childhood

Delirium vs. Dementia

Delirium: Disturbance in attention Abrupt onset with periods of lucidity Disorganized thinking Poor executive functioning Disorientation Anxiety and agitation Poor recall Delusions and hallucinations Dementia: Progressive deterioration of cognitive functioning and global impairment of intellect Difficulty with memory, problem solving, and complex attention Disturbances in executive functioning Aphasia: Loss of language ability Apraxia: Loss of purposeful movement Agnosia: Loss of sensory ability to recognize objects

Assess the risk for violence:

Does the patient have a wish or intent to harm? Does the patient have a plan? Does the patient have means available to carry out the plan? Does the patient have demographic risk factors (i.e., male gender, age 14 to 24 years, low socioeconomic status, inadequate support system, prison time)?

Third Generation

Dopamine system stabilizers May improve positive and negative symptoms and cognitive function Little risk of EPS or tardive dyskinesia

A woman is admitted to the critical care unit in acute respiratory distress. Her respiratory rate is 38 breaths/min and labored. Lung auscultation reveals crackles in the lung bases. CT scan results indicate a high probability of pulmonary embolism. Which of the following is the most frequently seen clinical indicator of pulmonary embolism? Dyspnea Hemoptysis Coughing Chest pain

Dyspnea Dyspnea, tachycardia, tachypnea, and anxiety are the most frequently seen clinical indicators of a pulmonary embolus. Chest pain also may occur but is not as likely as dyspnea.

The nurse uses many precautions during IV administration of vesicant chemotherapy agents primarily to prevent: Septicemia Extravasation Catheter occlusion Anaphylactic shock.

Extravasation One of the major concerns with the IV administration of vesicant chemotherapy agents is infiltration or extravasation of drugs into tissue surrounding the infusion site. When infiltrated into the skin, vesicants cause pain, severe local tissue breakdown, and necrosis. Specific measures to ensure adequate dilution, patency, and early detection of extravasation and treatment are important. The other options are not related to the administration of vesicants.

A patient was admitted to the emergency department with status epilepticus. Which of the following drugs would most likely be prescribed initially to stop the seizures? Phenytoin Fosphenytoin Lorazepam Phenobarbital

Lorazepam The initial goal of stopping the seizure is best accomplished through the use of a benzodiazepine. The first choice is usually lorazepam, but diazepam (Valium) may be given has a shorter half-life. Lorazepam crosses the blood-brain barrier more quickly than phenytoin or phenobarbital. Phenytoin or fosphenytoin are used for status epilepticus but generally after benzodiazepines. Phenobarbital typically is used only after a benzodiazepine or phenytoin has failed to control status epilepticus.

A patient with a lung mass found on chest x-ray is undergoing further testing. The nurse explains that a definitive diagnosis of lung cancer can be confirmed using which diagnostic test: Lung biopsy Pulmonary angiography Lung tomograms Computed tomography (CT) scans

Lung biopsy Although chest x-rays, lung tomograms, CT scans, MRI, and positron emission tomography (PET) can identify tumors and masses, a definitive diagnosis of a lung cancer requires identification of malignant cells in a biopsy or cytologic study of bronchial washings.

A patient is admitted to the critical care unit following an ischemic stroke. Which of the following sets of mean arterial pressure (MAP) and intracranial pressure (ICP) would have impairment in autoregulation? MAP 70 mm Hg; ICP 10 mm Hg MAP 80 mm Hg; ICP 20 mm Hg MAP 75 mm Hg; ICP 30 mm Hg MAP 100 mm Hg; ICP 15 mm Hg

MAP 75 mm Hg; ICP 30 mm Hg A cerebral perfusion pressure (CPP) of at least 60 mm Hg is required to maintain autoregulation. CPP = MAP - ICP so only option c would result in an inadequate CPP.

When comparing the usual initial blood glucose levels seen in diabetic ketoacidosis (DKA) and hyperosmolar hyperglycemic syndrome (HHS), the emergency department nurse would expect: The blood glucose in DKA and HHS to be equally elevated The blood glucose in DKA to be higher than the blood glucose in HHS The blood glucose in HHS to be higher than the blood glucose in DKA Neither DKA nor HHS affect blood glucose levels

The blood glucose in HHS to be higher than the blood glucose in DKA

An arterial blood gas report indicates that pH is 7.25, PaCO 2 is 30 mm Hg, and HCO 3 is 16 mEq/L (16 mmol/L). The nurse identifies which patient is most likely to exhibit these results? a. A 24-year-old with diabetic ketoacidosis b. A 17-year-old with panic attacks c. A 65-year-old with advanced emphysema d. A 54-year-old with vomiting

a. A 24-year-old with diabetic ketoacidosis

A 50-year-old patient sustained a rattlesnake bite 6 hours ago. He is having a severe reaction to the venom and antivenom is being flown to the hospital. The patient is bleeding from his IV insertion sites and continues to seep blood from the wound. Treatment for disseminated intravascular coagulation in this particular case would include which of the following? a. Clotting factors and antivenom b. Clotting factors and heparin c. IV fluids and antivenom d. IV fluids and heparin

a. Clotting factors and antivenom

A key aspect of teaching for the patient on anticoagulant therapy includes which instructions? a. Monitor for and report any signs of bleeding. b. Do not take acetaminophen (Tylenol) for a headache. c. Decrease your dietary intake of foods containing vitamin K. d. Arrange to have blood drawn twice a week to check drug effects.

a. Monitor for and report any signs of bleeding.

Which events cause increased ICP (select all that apply)? a. Necrotic cerebral tissue b. Edema from initial brain insult c. Blood vessel compression d. Vasodilation e. Brainstem compression and herniation

a. Necrotic cerebral tissue b. Edema from initial brain insult d. Vasodilation

On admission to the intensive care unit, a patient is diagnosed with compensated metabolic acidosis. During the assessment, what is the nurse most likely to identify? a. deep, rapid respirations b. Tachycardia and dysrhythmias c. Muscle twitching d. mental instability

a. deep, rapid respirations

Common psychosocial reactions of the stroke patient to the stroke include (select all that apply) a. depression. b. disassociation. c. intellectualization. d. sleep disturbances. e. denial of severity of stroke.

a. depression. d. sleep disturbances. e. denial of severity of stroke.

A patient is suspected of having a brain tumor. The signs and symptoms include memory deficits, visual disturbances, weakness of right upper and lower extremities, and personality changes. The nurse recognizes that the tumor is most likely located in the a. frontal lobe. b. parietal lobe. c. occipital lobe. d. temporal lobe.

a. frontal lobe.

The lungs act as an acid-base buffer by a. increasing respiratory rate and depth when CO2 levels in the blood are high, reducing acid load. b. increasing respiratory rate and depth when CO2 levels in the blood are low, reducing base load. c. decreasing respiratory rate and depth when CO2 levels in the blood are high, reducing acid load. d. decreasing respiratory rate and depth when CO2 levels in the blood are low, increasing acid load

a. increasing respiratory rate and depth when CO2 levels in the blood are high, reducing acid load.

A patient has a nursing diagnosis of risk for ineffective cerebral tissue perfusion related to cerebral edema. What is an appropriate nursing intervention for the patient? a. Perform frequent suctioning to prevent accumulation of respiratory secretions. b. Avoid positioning the patient with neck and hip flexion. c. Maintain hyperventilation to a PaCO2 of 15 to 20 mm Hg. d. Maintain cerebral perfusion pressure (CPP) < 60 mm Hg.

b. Avoid positioning the patient with neck and hip flexion.

During admission of a patient with a severe head injury to the emergency department, the nurse places the highest priority on assessment for a. patency of airway. b. presence of a neck injury. c. neurologic status with the Glasgow Coma Scale. d. cerebrospinal fluid leakage from the ears or nose.

a. patency of airway.

During the acute phase of a stroke, the nurse assesses the patient's vital signs and neurologic status at least every 4 hours. What is a cardiovascular sign that the nurse would see as the body attempts to increase cerebral blood flow? a. cardiac mumurs b. Hypertension c. low mean arterial pressure d. cardiac dysrhythmias

b. Hypertension

A patient has been diagnosed with occipital lobe damage after a car accident. What will the nurse anticipate the patient needs help with? a. Being able to feel heat b. Processing visual images c. Identifying smells appropriately d. Being able to say what she means

b. Processing visual images

Which of the following is the best diagnostic procedure for deep venous thrombosis (DVT)? a. D-dimer assay b. Venous duplex ultrasound c. Venography d. Complete blood count

b. Venous duplex ultrasound

A patient is seen in the emergency department after diving into the pool and hitting the bottom with a blow to the face that hyperextended the neck and scraped the skin off the nose. The patient also described "having double vision" when looking down. During the neurologic exam, the nurse finds the patient is unable to abduct either eye. The nurse recognizes this finding is related to a. a basal skull fracture. b. a stretch injury to bilateral CN VI. c. a stiff neck from the hyperextension injury. d. facial swelling from the scrape on the bottom of the pool.

b. a stretch injury to bilateral CN VI

Vasogenic cerebral edema increases intracranial pressure by a. shifting fluid in the gray matter. b. altering the endothelial lining of cerebral capillaries. c. leaking molecules from the intracellular fluid to the capillaries. d. altering the osmotic gradient flow into the intravascular component.

b. altering the endothelial lining of cerebral capillaries.

A nursing measure that is indicated to reduce the potential for seizures and increased intracranial pressure in the patient with bacterial meningitis is a. administering codeine for relief of head and neck pain. b. controlling fever with prescribed drugs and cooling techniques. c. keeping the room dark and quiet to minimize environmental stimulation. d. maintaining the patient on strict bed rest with the head of the bed slightly elevated.

b. controlling fever with prescribed drugs and cooling techniques.

DIC is a disorder in which a. the coagulation pathway is genetically altered, leading to thrombus formation in all major blood vessels. b. an underlying disease depletes hemolytic factors in the blood, leading to diffuse thrombotic episodes and infarcts. c. a disease process stimulates coagulation processes with resultant thrombosis, as well as depletion of clotting factors, leading to diffuse clotting and hemorrhage. d. an inherited predisposition causes a deficiency of clotting factors that leads to overstimulation of coagulation processes in the vasculature.

c. a disease process stimulates coagulation processes with resultant thrombosis, as well as depletion of clotting factors, leading to diffuse clotting and hemorrhage.

For a patient who is suspected of having a stroke, one of the most important pieces of information that the nurse can obtain is a. time of the patient's last meal. b. time at which stroke symptoms first appeared. c. patient's hypertension history and management. d. family history of stroke and other cardiovascular diseases.

b. time at which stroke symptoms first appeared.

A nurse is assessing a patient with a diagnosis of kidney failure for clinical indicators of metabolic acidosis. What should the nurse conclude is the reason metabolic acidosis develops with kidney failure? a. Depressed respiratory rate due to metabolic wastes, causing carbon dioxide retention b. Impaired glomerular filtration, causing retention of sodium and metabolic waste products c. Inability of the renal tubules to secrete hydrogen ions and conserve bicarbonate d. Inability of the renal tubules to reabsorb water to dilute the acid contents of blood

c. Inability of the renal tubules to secrete hydrogen ions and conserve bicarbonate

What are characteristics of the phosphate buffer system? a. Free acid radicals dissociate into H+ and CO2, buffering excess base b. Shifts chloride in and out of red blood cells in exchange for sodium bicarbonate, buffering both acids and bases c. Neutralizes a strong base to a weak base and water; resultant sodium biphosphate is eliminated by kidneys d. Neutralizes a strong acid to yield sodium biphosphate, a weak acid, and salt

c. Neutralizes a strong base to a weak base and water; resultant sodium biphosphate is eliminated by kidneys

The nurse on the clinical unit is assigned to four patients. Which patient should she assess first? a. Patient with a skull fracture whose nose is bleeding b. Older patient with a stroke who is confused and whose daughter is present c. Patient with meningitis who is suddenly agitated and reporting a headache of 10 on a 0-to-10 scale d. Patient who had a craniotomy for a brain tumor and who is now 3 days postoperative and has had continued vomiting

c. Patient with meningitis who is suddenly agitated and reporting a headache of 10 on a 0-to-10 scale

Nursing management of a patient with a brain tumor includes (select all that apply) a. discussing with the patient methods to control inappropriate behavior. b. using diversion techniques to keep the patient stimulated and motivated. c. assisting and supporting the family in understanding any changes in behavior. d. limiting self-care activities until the patient has regained maximum physical functioning. e. planning for seizure precautions and teaching the patient and the caregiver about antiseizure drugs.

c. assisting and supporting the family in understanding any changes in behavior. e. planning for seizure precautions and teaching the patient and the caregiver about antiseizure drugs.

The factor related to cerebral blood flow that most often determines the extent of cerebral damage from a stroke is the a. amount of cardiac output. b. O2 content of the blood. c. degree of collateral circulation. d. level of CO2 in the blood.

c. degree of collateral circulation.

The nurse is alerted to a possible acute subdural hematoma in the patient who a. has a linear skull fracture crossing a major artery. b. has focal symptoms of brain damage with no recollection of a head injury. c. develops decreased level of consciousness and a headache within 48 hours of a head injury. d. has an immediate loss of consciousness with a brief lucid interval followed by decreasing level of consciousness.

c. develops decreased level of consciousness and a headache within 48 hours of a head injury.

A patient is exhibiting word finding difficulty and weakness in his right arm. What area of the brain is most likely involved? a. brainstem. b. vertebral artery. c. left middle cerebral artery. d. right middle cerebral artery.

c. left middle cerebral artery.

An arterial blood gas report indicates the patient's pH is 7.25, PaCO 2 is 56 mm Hg, PaO 2 is 70, and HCO 3 is 20 mEq/L. Which disturbance should the nurse identify based on these results? a. Metabolic Alkalosis b. Respiratory Alkalosis c. Metabolic Acidosis d. Respiratory Acidosis

d. Respiratory Acidosis

The nurse explains to the patient with a stroke who is scheduled for angiography that this test is used to determine the a. presence of increased ICP. b. site and size of the infarction. c. patency of the cerebral blood vessels. d. presence of blood in the cerebrospinal fluid.

c. patency of the cerebral blood vessels.

A patient experiencing TIAs is scheduled for a carotid endarterectomy. The nurse explains that this procedure is done to a. decrease cerebral edema. b. reduce the brain damage that occurs during a stroke in evolution. c. prevent a stroke by removing atherosclerotic plaques blocking cerebral blood flow. d. provide a circulatory bypass around thrombotic plaques obstructing cranial circulation.

c. prevent a stroke by removing atherosclerotic plaques blocking cerebral blood flow.

A patient with diabetes has a serum glucose level of 824 mg/dL (45.7 mmol/L) and is unresponsive. After assessing the patient, the nurse suspects diabetic ketoacidosis rather than hyperosmolar hyperglycemic syndrome based on the finding of a. polyuria. b. severe dehydration. c. rapid, deep respirations. d. decreased serum potassium.

c. rapid, deep respirations.

A patient with intracranial pressure monitoring has a pressure of 12 mm Hg. The nurse understands that this pressure reflects a. a severe decrease in cerebral perfusion pressure. b. an alteration in the production of cerebrospinal fluid. c. the loss of autoregulatory control of intracranial pressure. d. a normal balance between brain tissue, blood, and cerebrospinal fluid.

d. a normal balance between brain tissue, blood, and cerebrospinal fluid.

In planning care and patient teaching for the patient with venous leg ulcers, the nurse recognizes that the most important intervention in healing and control of this condition is a. discussing activity guidelines. b. using moist environment dressings. c. taking horse chestnut seed extract daily. d. applying graduated compression stockings.

d. applying graduated compression stockings.

The recommended treatment for an initial VTE in an otherwise healthy person with no significant co-morbidities would include a. IV argatroban (Acova) as an inpatient. b. IV unfractionated heparin as an inpatient. c. subcutaneous unfractionated heparin as an outpatient. d. subcutaneous low-molecular-weight heparin as an outpatient.

d. subcutaneous low-molecular-weight heparin as an outpatient.

Information provided by the patient that would help differentiate a hemorrhagic stroke from a thrombotic stroke includes a. sensory disturbance. b. a history of hypertension. c. presence of motor weakness. d. sudden onset of severe headache.

d. sudden onset of severe headache.


Ensembles d'études connexes

Immunology Practice Questions_MCQ

View Set

3.6 Polynomial & Rational Inequalities

View Set

Exercise 6: Classification of Tissues

View Set

Macro Test 2. Chapter 7.a and Checkpoint 4

View Set

English 3 Vocab austere-vulnerable Definitions

View Set

Chapter 21: Activation of Cytotoxic T Cells - 4 functions of cytokines

View Set

Ch. 58 - Independent Contractor Agreements - Quiz Quesitons

View Set